Showing the sum of this telescoping series

Click For Summary
SUMMARY

The series \sum_{i=1}^{\infty} \frac{6}{9i^{2}+6i-8} is analyzed for convergence using partial fraction decomposition, resulting in the expression \frac{1}{3i-2} - \frac{1}{3i+4}. The divergence test is inconclusive, but the series can be expressed in a telescoping form, where most terms cancel out, leaving only 1 and \frac{1}{4} as non-canceling terms. The general term for the nth sum is S_{n} = \left( 1-\frac{1}{7} \right) + \left( \frac{1}{4} - \frac{1}{10} \right) + \dots + \left( \frac{1}{3n-2} - \frac{1}{3n+4} \right), confirming the series converges.

PREREQUISITES
  • Understanding of series convergence tests
  • Knowledge of partial fraction decomposition
  • Familiarity with telescoping series
  • Basic algebraic manipulation skills
NEXT STEPS
  • Study the properties of telescoping series in depth
  • Learn advanced techniques for series convergence tests
  • Explore applications of partial fraction decomposition in calculus
  • Investigate the behavior of series with polynomial denominators
USEFUL FOR

Students studying calculus, particularly those focusing on series convergence, mathematicians interested in series analysis, and educators teaching advanced algebra and calculus concepts.

Euler2718
Messages
90
Reaction score
3

Homework Statement



Determine whether each of the following series is convergent or divergent. If the series is convergent, find its sum

\sum_{i=1}^{\infty} \frac{6}{9i^{2}+6i-8}

Homework Equations



Partial fraction decomposition

\frac{1}{3i-2} - \frac{1}{3i+4}

The Attempt at a Solution



The divergence test is inconclusive, so I wrote as partial fractions and started analysing the nth sum:

S_{n} = \left( 1-\frac{1}{7} \right) + \left( \frac{1}{4} - \frac{1}{10} \right) + \left( \frac{1}{7} - \frac{1}{13} \right) + \left( \frac{1}{10} - \frac{1}{16} \right) + \dots

1 and 1/4 are the only terms that do not cancel, but how do I show this in the nth case? I'm having trouble writing it generally.
 
Physics news on Phys.org
Morgan Chafe said:

Homework Statement



Determine whether each of the following series is convergent or divergent. If the series is convergent, find its sum

\sum_{i=1}^{\infty} \frac{6}{9i^{2}+6i-8}

Homework Equations



Partial fraction decomposition

\frac{1}{3i-2} - \frac{1}{3i+4}

The Attempt at a Solution



The divergence test is inconclusive, so I wrote as partial fractions and started analysing the nth sum:

S_{n} = \left( 1-\frac{1}{7} \right) + \left( \frac{1}{4} - \frac{1}{10} \right) + \left( \frac{1}{7} - \frac{1}{13} \right) + \left( \frac{1}{10} - \frac{1}{16} \right) + \dots

1 and 1/4 are the only terms that do not cancel, but how do I show this in the nth case? I'm having trouble writing it generally.
Include the general term in your expansion:
##S_{n} = \left( 1-\frac{1}{7} \right) + \left( \frac{1}{4} - \frac{1}{10} \right) + \left( \frac{1}{7} - \frac{1}{13} \right) + \left( \frac{1}{10} - \frac{1}{16} \right) + \dots + \left( \frac{1}{3n-2} - \frac{1}{3n+4} \right) + \dots##
If you add in the term before and the one after the last term I wrote above, you should see how the telescoping happens.
 
  • Like
Likes Euler2718
Morgan Chafe said:

Homework Statement



Determine whether each of the following series is convergent or divergent. If the series is convergent, find its sum

\sum_{i=1}^{\infty} \frac{6}{9i^{2}+6i-8}

Homework Equations



Partial fraction decomposition

\frac{1}{3i-2} - \frac{1}{3i+4}

The Attempt at a Solution



The divergence test is inconclusive, so I wrote as partial fractions and started analysing the nth sum:

S_{n} = \left( 1-\frac{1}{7} \right) + \left( \frac{1}{4} - \frac{1}{10} \right) + \left( \frac{1}{7} - \frac{1}{13} \right) + \left( \frac{1}{10} - \frac{1}{16} \right) + \dots

1 and 1/4 are the only terms that do not cancel, but how do I show this in the nth case? I'm having trouble writing it generally.

Show that for every term of the form ##1/(2n)## there will be another term ##-1/(2n)## (corresponding to just two possible values of ##i##) and for every term ##1/(2n+1)## there is a cancelling term ##-1/(2n+1)##---again, corresponding to exactly two values of ##i##.
 
  • Like
Likes Euler2718
Ray Vickson said:
Show that for every term of the form ##1/(2n)## there will be another term ##-1/(2n)## (corresponding to just two possible values of ##i##) and for every term ##1/(2n+1)## there is a cancelling term ##-1/(2n+1)##---again, corresponding to exactly two values of ##i##.
Mark44 said:
Include the general term in your expansion:
##S_{n} = \left( 1-\frac{1}{7} \right) + \left( \frac{1}{4} - \frac{1}{10} \right) + \left( \frac{1}{7} - \frac{1}{13} \right) + \left( \frac{1}{10} - \frac{1}{16} \right) + \dots + \left( \frac{1}{3n-2} - \frac{1}{3n+4} \right) + \dots##
If you add in the term before and the one after the last term I wrote above, you should see how the telescoping happens.

Thanks guys, I got it now.
 
Question: A clock's minute hand has length 4 and its hour hand has length 3. What is the distance between the tips at the moment when it is increasing most rapidly?(Putnam Exam Question) Answer: Making assumption that both the hands moves at constant angular velocities, the answer is ## \sqrt{7} .## But don't you think this assumption is somewhat doubtful and wrong?

Similar threads

  • · Replies 7 ·
Replies
7
Views
2K
  • · Replies 1 ·
Replies
1
Views
2K
  • · Replies 1 ·
Replies
1
Views
2K
  • · Replies 5 ·
Replies
5
Views
2K
  • · Replies 2 ·
Replies
2
Views
2K
  • · Replies 3 ·
Replies
3
Views
2K
  • · Replies 2 ·
Replies
2
Views
2K
  • · Replies 2 ·
Replies
2
Views
1K
Replies
29
Views
2K
  • · Replies 7 ·
Replies
7
Views
2K